Twelve years ago and again five years ago, there were extend

This topic has expert replies
Legendary Member
Posts: 1574
Joined: Fri Jan 28, 2011 2:52 am
Thanked: 88 times
Followed by:13 members
Twelve years ago and again five years ago, there were extended periods when the Darfir Republic's currency,the pundra, was weak: its value was unusually low relative to the world's most stable currencies. Both times a weak pundra made Darfir's manufactured products a bargain on world markets, and Darfir's exports were up substantially. Now some politicians are saying that, in order to cause another similarly sized increase in exports, the government should allow the pundra to become weak again.

Which of the following, if true,provides the government with the strongest grounds to doubt that the politicians' recommendation, if followed, will achieve its aim?

A. Several of the politicians now recommending that the pundra be allowed to become weak made that same recommendation before each of the last two periods of currency weakness.
B. After several decades of operating well below peak capacity, Darfir's manufacturing sector is now operating at near-peak levels.
C. The economy of a country experiencing a rise in exports will become healthier only if the country's currency is strong or the rise in exports is significant.
D. Those countries whose manufactured products compete with Darfir's on the world market all currently have stable currencies.
E. A sharp improvement in the efficiency of Darfir's manufacturing plants would make Darfir's products a bargain on world markets even without any weakening of the pundra relative to other currencies.

[spoiler]OA: will be posted later. Please discuss each answer choice in detail[/spoiler]

User avatar
Legendary Member
Posts: 965
Joined: Thu Jan 28, 2010 12:52 am
Thanked: 156 times
Followed by:34 members
GMAT Score:720

by vineeshp » Sun May 01, 2011 11:11 am
I go for E.

This is a cause and effect reasoning Error.
Weak Pundra caused increase in exports (Which was caused by the products being a bargain).

So we need another reason that will help the exports.
That is provided by answer choice E.

A. Several of the politicians now recommending that the pundra be allowed to become weak made that same recommendation before each of the last two periods of currency weakness. - no reason to doubt it will fail this time.
B. After several decades of operating well below peak capacity, Darfir's manufacturing sector is now operating at near-peak levels. - still does not explain how to increase exports.
C. The economy of a country experiencing a rise in exports will become healthier only if the country's currency is strong or the rise in exports is significant. - target is to improve exports.
D. Those countries whose manufactured products compete with Darfir's on the world market all currently have stable currencies. irrelevant to discuss their currencies.
Vineesh,
Just telling you what I know and think. I am not the expert. :)

Legendary Member
Posts: 586
Joined: Tue Jan 19, 2010 4:38 am
Thanked: 31 times
Followed by:5 members
GMAT Score:730

by rohu27 » Sun May 01, 2011 4:55 pm
aspirant2011 wrote:Twelve years ago and again five years ago, there were extended periods when the Darfir Republic's currency,the pundra, was weak: its value was unusually low relative to the world's most stable currencies. Both times a weak pundra made Darfir's manufactured products a bargain on world markets, and Darfir's exports were up substantially. Now some politicians are saying that, in order to cause another similarly sized increase in exports, the government should allow the pundra to become weak again.

Which of the following, if true,provides the government with the strongest grounds to doubt that the politicians' recommendation, if followed, will achieve its aim?

A. Several of the politicians now recommending that the pundra be allowed to become weak made that same recommendation before each of the last two periods of currency weakness.
this if any strethtnes the argument tht poloticians made a similar comment below and economy did improve
B. After several decades of operating well below peak capacity, Darfir's manufacturing sector is now operating at near-peak levels.
this says tht the economy is alreayd at near peak levels,so no matter wht measures are taken (weakening of pundra) it will have no effect on the economy.correct answer.
C. The economy of a country experiencing a rise in exports will become healthier only if the country's currency is strong or the rise in exports is significant.
strenghtens the claim of politicnas
D. Those countries whose manufactured products compete with Darfir's on the world market all currently have stable currencies.
doesnt make us to doubt the plotician sstatemnt.they have stable currencies and still competing, may be by declinign the currency dafir can become more comeptitive? we dnt knw
E. A sharp improvement in the efficiency of Darfir's manufacturing plants would make Darfir's products a bargain on world markets even without any weakening of the pundra relative to other currencies.
this gives an alternate plan to make darfirs products a bargain BUT dnt forget wht we are out to prove, we need to show ploticians argument is not strong, there may well be other routes via which the economy can be strong, we are not bothered.

B offers that explanation, already full economy so no point implemeting naythng.

[spoiler]OA: will be posted later. Please discuss each answer choice in detail[/spoiler]

Newbie | Next Rank: 10 Posts
Posts: 6
Joined: Fri Apr 15, 2011 5:32 am
Thanked: 2 times
Thanks for the explanation
Last edited by mymbadreamz on Sat Jan 09, 2021 11:21 am, edited 2 times in total.
.

Legendary Member
Posts: 768
Joined: Mon Nov 30, 2009 3:46 am
Thanked: 21 times
Followed by:7 members

by GMATMadeEasy » Mon May 02, 2011 4:15 am
@vineeshp : If you see reasoning used carefully, politicains didn't say that it is the only reason to raise export. Instead , he has assumed that a situation that worked in the past , wll work in the present or in the future. What if there is a circumstance that does not allow so to happen the same way ?

Go ahead and bring down the currency.. demand will increase.. then what.. all plants are running at their peak capacity.

If E wer used by an oppsotion politician :) , the politician will be quick to say; hey why not to use both and have cumulative effect.

Legendary Member
Posts: 1574
Joined: Fri Jan 28, 2011 2:52 am
Thanked: 88 times
Followed by:13 members

by aspirant2011 » Mon May 02, 2011 8:55 am
thanks rohu, yup the answer is B ..........

User avatar
Master | Next Rank: 500 Posts
Posts: 232
Joined: Fri Jun 18, 2010 7:09 am
Thanked: 1 times
Followed by:2 members

by ruplun » Sat Jun 08, 2013 8:50 pm
rohu27 wrote:
aspirant2011 wrote:Twelve years ago and again five years ago, there were extended periods when the Darfir Republic's currency,the pundra, was weak: its value was unusually low relative to the world's most stable currencies. Both times a weak pundra made Darfir's manufactured products a bargain on world markets, and Darfir's exports were up substantially. Now some politicians are saying that, in order to cause another similarly sized increase in exports, the government should allow the pundra to become weak again.

Which of the following, if true,provides the government with the strongest grounds to doubt that the politicians' recommendation, if followed, will achieve its aim?

A. Several of the politicians now recommending that the pundra be allowed to become weak made that same recommendation before each of the last two periods of currency weakness.
this if any strethtnes the argument tht poloticians made a similar comment below and economy did improve
B. After several decades of operating well below peak capacity, Darfir's manufacturing sector is now operating at near-peak levels.
this says tht the economy is alreayd at near peak levels,so no matter wht measures are taken (weakening of pundra) it will have no effect on the economy.correct answer.
C. The economy of a country experiencing a rise in exports will become healthier only if the country's currency is strong or the rise in exports is significant.
strenghtens the claim of politicnas
D. Those countries whose manufactured products compete with Darfir's on the world market all currently have stable currencies.
doesnt make us to doubt the plotician sstatemnt.they have stable currencies and still competing, may be by declinign the currency dafir can become more comeptitive? we dnt knw
E. A sharp improvement in the efficiency of Darfir's manufacturing plants would make Darfir's products a bargain on world markets even without any weakening of the pundra relative to other currencies.
this gives an alternate plan to make darfirs products a bargain BUT dnt forget wht we are out to prove, we need to show ploticians argument is not strong, there may well be other routes via which the economy can be strong, we are not bothered.

B offers that explanation, already full economy so no point implemeting naythng.

[spoiler]OA: will be posted later. Please discuss each answer choice in detail[/spoiler]

in weakening we are told that:

d. Even if the choice introduces new information, you need to accept them as correct choices, if they are weakening the conclusion.

so why E is incorrect.Please explain

Junior | Next Rank: 30 Posts
Posts: 18
Joined: Wed Jul 01, 2015 11:22 pm
Thanked: 1 times
GMAT Score:720

by mack13 » Tue Jul 05, 2016 9:49 am
Hi,

Why do we have to assume that to satisfy the demand for exports we need to manufacture more ?
I realised that supply might still be more as compared to the demand and thus eliminated answer choice B
Where did I go wrong here ?
Please take a moment to hit Thanks if you like my post. :)

Senior | Next Rank: 100 Posts
Posts: 39
Joined: Sat Jan 12, 2019 1:08 am

by meenakshimiyer » Sat Jan 12, 2019 9:48 pm
The argument states that during periods of weak currency (lower value vs other countries), X experience a sharp INCREASE in exports. Politicians are suggesting that the country intentionally weaken the currency so X can experience a similar INCREASE in exports.

The correct answer to this 'weaken' question needs to ATTACK a necessary assumption to the politicians plan.

A. If anything, this answer strengthens the position of the politicians
B. This answer choice (correct) states that manufacturing is already at peak levels. That means that even if the currency become devalued and demand for products increases, X would not be able to increase manufacturing to meet that demand. The conclusion of the politicians is completely weakened (if not destroyed). [Note: the assumption you are attacking here is as follows: Politicians are assuming that X is able to increase production to meet the increase in demand that would result from a devalued currency.]
C. Out of scope answer
D. Out of scope (or perhaps even a little strengthening)
E. Out of scope
Hence, B is the answer.